1
$\begingroup$

I'm trying to solve the following matrix calculus problem:

$\text{argmin}_{v \in R_+^K}(v'\Sigma v) \hspace{0.5pc} \text{subject to} \hspace{0.5pc} 1'v=1$

where $\Sigma$ is a well-behaved (symmetric, positive-definite) kxk variance-covariance matrix and $1'$ is a 1xk vector of ones.

$\endgroup$
1
  • 1
    $\begingroup$ Are you using $a'$ to denote $a$ transpose? a^t may be easier to interpret. Also, this question may be more appropriate for math.stackexchange.com $\endgroup$ Commented Oct 19, 2022 at 20:54

0

Start asking to get answers

Find the answer to your question by asking.

Ask question

Explore related questions

See similar questions with these tags.